Difference between revisions of "1983 AHSME Problems/Problem 7"

(Created page with "== Problem== Alice sells an item at \$<math>10</math> less than the list price and receives <math>10\%</math> of her selling price as her commission. Bob sells the same item...")
 
(Slightly cleaned up the solution)
Line 1: Line 1:
 
== Problem==
 
== Problem==
  
Alice sells an item at \$<math>10</math> less than the list price and receives <math>10\%</math> of her selling price as her commission.  
+
Alice sells an item at <math>\$10</math> less than the list price and receives <math>10\%</math> of her selling price as her commission.  
Bob sells the same item at \$<math>20</math> less than the list price and receives <math>20\%</math> of his selling price as his commission.  
+
Bob sells the same item at <math>\$20</math> less than the list price and receives <math>20\%</math> of his selling price as his commission.  
If they both get the same commission, then the list price in dollars is
+
If they both get the same commission, then the list price is
  
<math>\textbf{(A) } 20\qquad
+
<math>\textbf{(A) } \$20\qquad
\textbf{(B) } 30\qquad
+
\textbf{(B) } \$30\qquad
\textbf{(C) } 50\qquad
+
\textbf{(C) } \$50\qquad
\textbf{(D) } 70\qquad
+
\textbf{(D) } \$70\qquad
\textbf{(E) } 100 </math>
+
\textbf{(E) } \$100 </math>
  
 
==Solution==
 
==Solution==
  
If <math>x</math> is the list price, then <math>10\%(x-10)=20\%(x-20)</math>. Solving this, you get <math>x=30</math>. So the answer is <math>\boxed{\textbf{(B) }30}</math>
+
If <math>x</math> is the list price, then <math>10\%(x-10)=20\%(x-20)</math>. Solving this equation gives <math>x=30</math>, so the answer is <math>\boxed{\textbf{(B) }\$30}</math>.

Revision as of 18:34, 26 January 2019

Problem

Alice sells an item at $$10$ less than the list price and receives $10\%$ of her selling price as her commission. Bob sells the same item at $$20$ less than the list price and receives $20\%$ of his selling price as his commission. If they both get the same commission, then the list price is

$\textbf{(A) } $20\qquad \textbf{(B) } $30\qquad \textbf{(C) } $50\qquad \textbf{(D) } $70\qquad \textbf{(E) } $100$

Solution

If $x$ is the list price, then $10\%(x-10)=20\%(x-20)$. Solving this equation gives $x=30$, so the answer is $\boxed{\textbf{(B) }$30}$.